Inscription / Connexion Nouveau Sujet
Niveau Licence Maths 1e ann
Partager :

une inégalité

Posté par
jb2017
17-07-17 à 17:15

bonjour
Je propose ce problème suivant :
On suppose que a^3+b^3+c^3=3; a,b,c \geq 0
Montrer que
a^4+b^+c^4+ 3(a+b+c) \leq 12

Méthode des multiplicateurs ou "à la main".

Posté par
Razes
re : une inégalité 17-07-17 à 18:11

Bonjour ,

La contrainte est : g(a,b,c)=a^3+b^3+c^3-3
On veut montrer que: f(a,b,c)=a^4+b^4+c^4+ 3(a+b+c) \leq 12

On applique le multiplicateur de Lagrange:

\triangledown f(a,b,c)=\begin{pmatrix}4a^3+3\\ 4b^3+3\\ 4c^3+3\end{pmatrix}; \triangledown g(a,b,c)=\begin{pmatrix}3a^2\\ 3b^2\\ 3c^2\end{pmatrix}

\exists \lambda \in\mathbb{R}; \triangledown f(a,b,c)-\lambda\triangledown g(a,b,c)=0

\left\{\begin{matrix}4a^3+3-\lambda 3a^2=0\\ 4b^3+3-\lambda 3b^2=0\\ 4c^3+3-\lambda 3c^2=0\end{matrix}\right.

Sommons.

Posté par
jb2017
re : une inégalité 17-07-17 à 18:28

@Rasez
Tout à fait d'accord mais je ne somme pas. On calcule les  3 égalités avec  \lambda  pour voir que cela implique a=b=c et c'est gagné. D'accord?

En fait on utilise ici de l'analyse. Est-ce qu'il n'y aurait pas une solution plus basique, i.e
où on utilise des   connaissances de niveau lycée?

Pour info, dans un post précédent où tu es intervenu, j'ai posé une inégalité non résolue,  trouvée sur math.net et qui semble difficile à démontrer.
Je pense avoir montré ( mais encore il faut que je mette cela propre) que le problème se ramène à cette inégalité ici.

Posté par
Razes
re : une inégalité 17-07-17 à 19:00

Ok.
Donc, tu as fait:
\lambda=a+\frac{1}{a^2} =b+\frac{1}{b^2}=c+\frac{1}{c^2}

As tu trouvé une seule solution a=b=c ou il y en d'autres?

Posté par
Razes
re : une inégalité 17-07-17 à 19:09

Pour l'autre sujet,  la methode du multiplicateur de Lagrange permettra surement de trouver la solution. Mais il y a beaucoup de calculs du fait de la presence du denominateur. Ceci sans compter les risques d'erreurs.

As tu lu ce que j'avais propose?

Posté par
larrech
re : une inégalité 17-07-17 à 19:10

Bonjour,

Une fois éliminée la solution a=b=c, on reste, sauf erreur, avec

4(a^2+b^2)=3(a+b)

et 2 autres équations obtenues par permutations circulaires.
Ne faudrait-il pas  essayer de poursuivre ??

Posté par
Razes
re : une inégalité 17-07-17 à 19:20

Si tu resouds les système, normalement tu trouve aussi:

a=\frac{b}{b-1} ainsi que pour c. Ceci te permettra de dire seulement que c'est un extremum. Est ce un maximum?

Posté par
jb2017
re : une inégalité 17-07-17 à 20:25

@Razes je pense que tu veux dire 4/3a+1/a^2  mais cela revient au même pour le raisonnement.
Donc si on pose f(a)=4/3a+1/a^2
On a f(a)=f(b)=f(c)  qui conduit à a=b=c. Cela n'est pas étonnant.
Mais bien sûr @larrech a raison,  on doit vérifier que cette équation n' a pas d'autres solutions à l'intérieur de la surface. C'est un peu de calcul (faisable) mais on n'a pas d'autre point critique  à l'intérieur.
Ce qui veut dire que les minimums sont atteints aux bords de la surface.
Par exemple (0,0,3^(1/3)) conduit à un minimum global. Mais ce n'est pas la question.
Il faut vérifier, pour terminer  si il n'y a pas un maximum absolu aux bords. Ce n'est pas le cas.
Pour le voir on pose par exemple a=0. On trouve par exemple
(0,\left(\frac{3}{2}\right)^{1/3},\left(\frac{3}{2}\right)^{1/3} comme maximum mais il est plus petit que 12 donc ce n'est pas un maximum global. Idem par permutation des (a,b,c)
Il s'agit de comprendre ici que la méthode des multiplicateurs donne la réponse ici avec des calculs faisable.
Mais on voit que malgré tout il faut du calculs que l'on ne peut pas reporter ici à cause de la longueur.
Pour l'autre problème je pense que tu veux dire est-ce que j'ai suivi cette méthode?
La réponse est oui mais j'ai fait les calculs à la machine (on a des polynômes j'ai donc calculer les résultants...)
Pour trouver un point critique j'ai été "réduit à trouver les valeurs de c dans l'intervalle
[0, 3^(1/3)] qui annule ce polynôme. Bien sûr j'ai retrouvé c=1 qui aurait conduit à a=b=1. Mais le polynôme est degré 225. Comme ici il y a des valeurs autre que 1 qui annulent le polynôme, elle sont incalculables exactement et c'est foutu.
C'est pour cela que dans ce genre de problème la recherche d'un extremum
doit se faire autrement qu'avec les multiplicateurs.
C'est pour cela que même si on a la réponse, je cherche une méthode "élémentaire"
(i.e utiliser tout l'arsenal des inégalités usuelles C-S, Holder,....).


  


  







Posté par
carpediem
re : une inégalité 17-07-17 à 21:06

salut

a, b et c sont positifs donc

s = a^4 + b^4 + c^4 + 3 (a + b + c) \le a^3(a + b+ c) + b^3(a + b+ c) + c^3(a + b+ c) + 3(a + b + c) = 6(a + b+ c)

or la fonction cube est croissante et convexe sur \R^+ donc

\dfrac 1 {18^3} s^3 \le \left( \dfrac {a + b + c} 3 \right)^3 \le \dfrac 1 3 (a^3 + b^3 +c^3) = 1

donc s \le 18


désolé j'ai pas mieux ...

Posté par
Razes
re : une inégalité 17-07-17 à 22:20

Razes @ 17-07-2017 à 19:00

Ok.
Donc, tu as fait:
\lambda=a+\dfrac{1}{a^2} =b+\dfrac{1}{b^2}=c+\dfrac{1}{c^2}

As tu trouvé une seule solution a=b=c ou il y en d'autres?

Désolé, je m'étais trompé:

a,b,c positifs non nuls.

\left\{\begin{matrix}4a^3+3-\lambda 3a^2=0\\ 4b^3+3-\lambda 3b^2=0\\ 4c^3+3-\lambda 3c^2=0\end{matrix}\right.\Leftrightarrow \lambda =\dfrac{4}{3}a+\dfrac{1}{a^2}=\dfrac{4}{3}b+\dfrac{1}{b^2}=\dfrac{4}{3}c+\dfrac{1}{c^2}

Prenons une des équations: \dfrac{4}{3}\left (a-b\right )+\dfrac{1}{a^2}-\dfrac{1}{b^2}=0\Leftrightarrow \left (a-b\right )\left (\dfrac{4}{3}-\dfrac{a+b}{a^2b^2}\right )=0\Leftrightarrow

\left (a-b\right )\left (4a^2b^2-3a-3b\right )=0
Donc, il y a la solution a=b mais aussi les deux solutions de l'équation du second degré en a ou en b

Posté par
Razes
re : une inégalité 18-07-17 à 15:47

Bonjour,
La majoration par 12 est facilement obtenu, on peut l'obtenir en majorant chaque groupe d'éléments à part.
a+b+c: On exploite, comme l'a proposé Carpediem, la convexite de la fonction cube. Ou en utilisant l'inégalité de Minkowski. On obtient: a+b+c=<3.

a^4+b^4+c^4: utilisons Chebychev, nous obtenons a^4+b^4+b^4=<\frac{1}{3}(a^2+b^2+c^2)^2.
Maintenant nous utilsons Minkowski pour majorer a^2+b^2+c^2 en choisissant p=\frac{3}{2} afin d'obtenir des cubes. Nous obtenons a^3+b^3+c^3=<3

En utilisant les deux résultats, le tour est joué.

Posté par
carpediem
re : une inégalité 18-07-17 à 20:13

il me semble que pour utiliser l'inégalité de Chebychev il faut des ""suites"" croissantes et/ou décroissantes ...

Posté par
jb2017
re : une inégalité 18-07-17 à 22:11

@Razes, effectivement comme le dit @carpediem avec  Cheb. il faut faire attention d'ailleurs avec (0,0,3) on voit que cela ne fonctionne pas.  
Les multiplicateurs ça marche. on est d'accord que l'on cherche une autre solution?  

Posté par
Razes
re : une inégalité 19-07-17 à 10:08

Bonjour,

@carpediem , @jb2017,

carpediem @ 18-07-2017 à 20:13

il me semble que pour utiliser l'inégalité de Chebychev il faut des ""suites"" croissantes et/ou décroissantes ...

Effectivement, le signe est inversé, et l'inégalité obtenue ne fait pas l'affaire a^4+b^4+b^4\geqslant \frac{1}{3}(a^2+b^2+c^2)^2.

Désolé.

Posté par
Alexique
re : une inégalité 19-07-17 à 18:39

Citation :
Les multiplicateurs ça marche. on est d'accord que l'on cherche une autre solution?  

Ben, il reste les solutions non triviales autres que a=b=c=1. Je me lance :
x \mapsto \frac43 x +\frac{1}{x^2} est décroissante stricte sur [0, \sqrt[3]{\frac32}] et croissante stricte sur   [\sqrt[3]{\frac32},3]. Par symétrie des rôles, supposons a \geq b \geq c. Alors
- soit a,b,c \leq \sqrt[3]{\frac32} et alors par injectivité, a=b=c(=1)
- soit c \leq \sqrt[3]{\frac32}, a,b \geq \sqrt[3]{\frac32} et alors par injectivité a=b. Mais a^3+b^3=2a^3 \geq 3 \implies c^3 \leq 0 \implies c=0 \implies (a,b,c) = (\sqrt[3]{\frac32},\sqrt[3]{\frac32},0) (c'est une solution mais alors est-ce qu'elle donne plus ou moins que 12 ?)
- soit a \geq \sqrt[3]{\frac32}, c,b \leq \sqrt[3]{\frac32} et alors b=c et la, on ne coupe pas au système a^3+2b^3=3 et 4a^2b^2-3(a+b)=0 (on trouve deux solutions graphiquement mais rien de prouver rigoureusement...) Du coup, il reste du travail...

Sans analyse, si on note p_3 = a^3+b^3+c^3=3 la 3eme somme de newton d'ordre 3, et e_1 = a+b+c, e_2 = ab+bc+ca, e_3=abc les fonctions symétriques élémentaires de a,b,c, on a (cf ) p_4+3p_1 = -\frac13 e_1^4+7e_1+2e_2^2. J'ai pour cela remplacer les sommes de Newton par leurs expressions par les fonctions symétriques élémentaires en utilisant également p_3=3 \implies e_3 = 1+e_1e_2-\frac{1}{3}e_1^3. Il reste à conclure par (je ne vois que ça) une inégalité de Mc Laurin... en tout cas, je n'ai pas fait de majoration jusque là donc pas de pertes d'information... En utilisant 3e_2 \leq e_1^2, ça ne marche pas par exemple et je n'ai pas d'autres idées...

En gros, je vous livre des 3/4 de solutions qui sont peut-être des quarts après réflexion. Éventuellement, récidiver et reposter sur mathématiques.net cette inégalité ainsi que nos réflexions, je pense que le plus gros est fait et que cela peut être fructueux cette fois !  

Posté par
Alexique
re : une inégalité 19-07-17 à 22:35

ok, je termine la méthode analytique :
- (a,b,c) = (\sqrt[3]{\frac32},\sqrt[3]{\frac32},0) conduit à une expression valant a^4+b^4+c^4+3(a+b+c)=2(\frac32)^{\frac43}+6(\frac32)^{\frac13} \approx 10,30 donc ok, c'est pas le max
- le système a^3+2b^3=3 et 4a^2b^2-3(a+b)=0 n'a en fait pas de solution (il y en a deux graphiquement mais sur \mathbb{R}^2). En effet, il suffit même de se contenter de regarder ce qu'il se passe dans le quart de plan a,b \geq 0. On a (a+b)^2 \geq 4ab (toujours vrai) et 4(ab)^2-3(a+b)=0 impliquent ab \geq \sqrt[3]{\frac94}. On a aussi par l'inégalité arithmético-géométrique \frac32 = \frac{a^3+2b^3}{2} \geq \sqrt{2a^3b^3} \implies ab \leq \sqrt[3]{\frac98} d'où l'encadrement absurde \sqrt[3]{\frac94}\leq ab \leq \sqrt[3]{\frac98} (si y'a plus simple,...).
Bref, seule solution : a=b=c=1 fournit un extremum valant 12 donc à un maximum.
Je crois que la méthode par analyse (extrema liés) est complètement achevée mais l'autre m'intéresse vraiment, je continue de plancher.

Peut-être peux-tu nous expliquer maintenant en quoi ce problème est une reformulation du problème évoqué ici Inequation ?

Posté par
jb2017
re : une inégalité 20-07-17 à 10:47

Bonjour @Alexique
Non c'est faux. En fait j'essaie (pour m'amuser) de résoudre le problème en question posé sur math.net, lui même pris sur un cite étranger (américain   je crois) et visiblement il semble difficile à résoudre puisque des personnes avec beaucoup d'idées échouent malgré tout. Evidemment j'ai essayé un peu tout et j'ai aussi exploité les idées des un et des autres. Mais j'ai transformé le problème en des problèmes équivalents ou analogues mais tout aussi difficile. Avec tout cela je me suis un peu mélangé les pinceaux et le problème ici n'est pas vraiment équivalent du moins je crois.
J'essaie de lever la difficulté du problème en question (je rappelle qui n'est pas symétrique) en trouvant une majoration par une forme symétrique.
Plus précisément on a sous la contrainte
a^3+b^3+c^3=3  (tout est positif)
f(a,b,c)=a^3(a+b)+b^3(b+c)+c^3(a+c)\leq 6 ( à démontrer)

Mais le problème avec
g(a,b,c)=a^3(b+c)+b^3(c+a)+c^3(a+b)\leq 6  est surement plus facile à démontrer vu la symétrie.    (maximum atteint en (a=b=c=1)

Mais si par hasard on avait f(a,b,c)\leq g(a,b,c) c'est gagné. Pour suivre cette idée il faut trouver la bonne fonction g.  
Bien sûr c'est une idée comme une autre.
On peut espérer sur ce forum  que quelqu'un apporte la solution surtout que la question est abordable par tout le monde.

Posté par
Alexique
re : une inégalité 20-07-17 à 11:10

Mais du coup, a^3(b+c)+b^3(c+a)+c^3(a+b)\leq 6 \iff 3(a+b+c)-(a^4+b^4+c^4) \leq 6  ce qui n'est pas exactement l'énoncé de ce topic... Cela dit, je doute que la résolution soit plus compliquée que tout ce que l'on vient de faire.

Posté par
jb2017
re : une inégalité 20-07-17 à 11:17

Oui c'est ce que je dit g(a,b,c)\leq 6c'est analogue à ce que j'ai posé et cela ne doit pas être un problème.
Mais attention je n'ai pas dit que f(a,b,c)\leq g(a,b,c)
En fait je n'en sais rien.  
f(a,b,c) c'est l'expression non symétrique et là f(a,b,c)\leq 6 c'est vraiment plus compliqué.

Posté par
nadiasoeur123
re : une inégalité 20-07-17 à 12:32

Bonjour ;

Comme on a : (a;b;c)\in\mathbb R^{*+3} tel que a^3+b^3+c^3=3 , procédons par disjonction de cas .

1) a; b et c sont tous inférieurs à 1 .

0<a<1 \Rightarrow 0<a^3<1
0<b<1 \Rightarrow 0<b^3<1
0<c<1 \Rightarrow 0<c^3<1

donc : 0<a^3+b^3+c^3<3

donc ce cas est à écarter .

2) a; b et c sont tous supérieurs à 1 .

 a>1 \Rightarrow a^3>1
 b>1 \Rightarrow b^3>1
 c>1 \Rightarrow c^3>1

donc :  a^3+b^3+c^3>3

donc ce cas est à écarter .

3) a = b = c = 1 .

a^4+b^4+c^4 + 3(a+b+c) = 12 .

4) a = b = 1 et c \ne 1 .

a^3+b^3+c^3 = 2+c^3 = 3 \Rightarrow c^3 = 1 \Rightarrow c = 1 ,

donc ce cas est à écarter .

De même pour : a = c = 1 et b \ne 1 et b = c = 1 et a \ne 1

5) a = 1 et (b et c) \ne 1 .

On a : b^3 + c^3 = 2 \Rightarrow c^3=2-b^3 > 0 \Rightarrow 0<b<\sqrt[3]{2} et c = \sqrt[3]{2-b^3} , donc :

a^4+b^4+c^4 + 3(a+b+c) = 1 + b^4 + (2-b^3)^{\frac{4}{3}} + 3(1+b+(2-b^3)^{\frac{1}{3}})

En étudiant la fonction f définie sur ]0;1[\cup ]1;\sqrt[3]{2}[ par ; f(x) = 1 + x^4 + (2-x^3)^{\frac{4}{3}} + 3(1+x+(2-x^3)^{\frac{1}{3}}) on trouve que f est majorée par 12  .

Conclusion :  \forall (a;b;c)\in\mathbb R^{+3} tel que a^3+b^3+c^3=3 on a :

a^4+b^4+c^4 + 3(a+b+c) \le 12 .

On a : a^4+b^4+c^4 + 3(a+b+c) = 12 pour a=b=c=1 .

Posté par
nadiasoeur123
re : une inégalité 20-07-17 à 12:37

re-bonjour ;

Suite à une remarque de jb2017 , il y a des cas qui manquent .

Posté par
Razes
re : une inégalité 20-07-17 à 12:42

Même remarques.

Posté par
Razes
re : une inégalité 20-07-17 à 19:13

Bonjour,

Appliquons l'inégalité de Hölder

\sum _{{k=1}}^{n}|x_{k}\ y_{k}|\leq \left(\sum _{{k=1}}^{n}|x_{k}|^{p}\right)^{{1/p}}\left(\sum _{{k=1}}^{n}|y_{k}|^{q}\right)^{{1/q}}; Avec:  1 \leqslant p, q \leqslant +\infty et \dfrac{1}{q} + \dfrac{1}{q} = 1

a) Élément a+b+c:
a+b+c=\left (a\times 1+b\times 1+c\times 1\right )\leqslant\left (a^3+b^3+c^3\right )^\frac{1}{3}.\left (1^\frac{3}{2}+1^\frac{3}{2}+1^\frac{3}{2}\right )^\frac{2}{3}=3^\frac{1}{3}.3^\frac{2}{3}=3

b) Élément a^4+b^4+c^4:
Soient \alpha ,\beta positifs tel que \alpha +\beta =4 et 1 \leqslant p, q \leqslant +\infty et \dfrac{1}{q} + \dfrac{1}{q} = 1

Donc:
a^4+b^4+c^4=a^{\alpha +\beta}+b^{\alpha +\beta}+c^{\alpha +\beta}\leqslant
 \\ \left ( (a^\alpha)^p+(b^\alpha)^p+(c^\alpha)^p\right )^\frac{1}{p}.\left ( (a^\beta)^q+(b^\beta)^q+(c^\beta)^q\right )^\frac{1}{q}=\left ( a^{p\alpha}+b^{p\alpha}+c^{p\alpha}\right )^\frac{1}{p}.\left ( a^{q\beta}+b^{q\beta}+c^{q\beta}\right )^\frac{1}{q}

Choisissons : p\alpha=3 et q\beta=1

Nous aurons donc: a^4+b^4+c^4\leqslant\left ( a^3+b^3+c^3\right )^\frac{1}{p}.\left ( a+b+c\right )^\frac{1}{q}

Déterminons donc, p, q, \alpha,\beta à partir des équations suivantes faciles à résoudre:

p\alpha=3; q\beta=1; \dfrac{1}{q} + \dfrac{1}{q} = 1;\alpha +\beta =4
Nous aurons à résoudre l'équation 3\beta^2-14\beta +12=0
Nous obtenons deux racines,  on en prends une:

\alpha =\dfrac{5-\sqrt{13}}{3}; \beta =\dfrac{7+\sqrt{13}}{3}; p=3\alpha =5-\sqrt{13};q=\beta =\dfrac{7+\sqrt{13}}{3}

Nous aurons donc: a^4+b^4+c^4\leqslant 3^\frac{1}{p}.3^\frac{1}{q}=3

c) Conclusion:

a^4+b^4+c^4+ 3(a+b+c) \leq 3+3*3=12

CQFD enfin

Posté par
jb2017
re : une inégalité 20-07-17 à 19:36

bonjour
@Razes , c'est bien et  ton idée me semble correcte mais je suis perdu avec ton équation avec \beta.
A mon avis c'est peut être bon ou bien il  y a une petite erreur mais   cela doit être corrigeable.

Posté par
Razes
re : une inégalité 20-07-17 à 19:51

jb2017 @ 20-07-2017 à 19:36

bonjour
@Razes , c'est bien et  ton idée me semble correcte mais je suis perdu avec ton équation avec \beta.
A mon avis c'est peut être bon ou bien il  y a une petite erreur mais   cela doit être corrigeable.
Bonjour,
Avant de parler d'erreur, essais de suivre les calculs.

Posté par
Razes
re : une inégalité 20-07-17 à 19:57

Peut-être une erreur, je corrige.

Posté par
Razes
re : une inégalité 20-07-17 à 20:14

J'ai trouvé un erreur dans mes calculs, désolé.

Posté par
Razes
re : une inégalité 05-08-17 à 03:39

Bonsoir,

a,b,c \geq 0; a^3+b^3+c^3=3; Montrer que : f(a,b,c)=a^4+b^4+c^4+ 3(a+b+c) \leq 12

Posons: x=a^3, y=b^3, z=c^3\Leftrightarrow a=\sqrt[3]{x};b=\sqrt[3]{y};c=\sqrt[3]{z};
Soient : \lambda _1=\lambda _2=\lambda _3=\frac{1}{3}\Rightarrow  \lambda _1+\lambda _2+\lambda _3=1

Posons: h(t)=t^{\frac{4}{3}}+3t^{\frac{1}{3}}, la fonction h est convexe.

f(a,b,c)=a^4+b^4+c^4+ 3(a+b+c) =h(x)+h(y)+h(z)

Convexité
\forall x_1,\cdots,x_p\in I\quad h\left(\dfrac{x_1+\cdots+x_p}p\right)\le\dfrac{h(x_1)+\cdots+h(x_p)}p

Donc:
h\left (\dfrac{x+y+z}{3} \right )\leqslant \dfrac{h(x)+h(y)+h(z)}{3}= \dfrac{f(a,b,c)}{3}\Leftrightarrow h\left (\dfrac{3}{3}\right )=h(1)\leqslant \dfrac{f(a,b,c)}{3}

C'est à dire: f(a,b,c)\geqslant 3h(1)=3\times 4=12

Donc l'énoncé présente une dans le symbole inégalité.

Posté par
Razes
re : une inégalité 05-08-17 à 03:40

oubliez les lambda_i

Posté par
Razes
re : une inégalité 05-08-17 à 04:02

L'autre ereur est que a,b,c>0

Posté par
Alexique
re : une inégalité 05-08-17 à 16:01

Et l'autre est que h est convexe... Mais celle-là est vraiment très embêtante... En plus, tu pars en disant :

Citation :
Montrons que f(a,b,c) 12
et tu conclus que
Citation :
f(a,b,c) 12
...

Je maintiens que tu devrais être plus prudent avant de poster et te remettre à peine plus en question.

Posté par
jb2017
2 inégalités 06-08-17 à 10:13

Bonjour
Voici deux inégalités à établir (à considérer comme une énigme)

1. Sous les  contraintes  a,b,c\geq 0 et a^3+b^3+c^3=3,  
trouver

\max (a^4+b^4+c^4)    

2. sous les mêmes contrainte montrer que

a^3b+b^3c+c^3a \leq 3

*** message déplacé ***

Posté par
malou Webmaster
re : une inégalité 06-08-17 à 10:29

attentionextrait de c_faq la FAQ du forum :

Q03 - Pourquoi ne faut-il pas faire du ''multi-post'' ?



Vous devez être membre accéder à ce service...

Pas encore inscrit ?

1 compte par personne, multi-compte interdit !

Ou identifiez-vous :


Rester sur la page

Inscription gratuite

Fiches en rapport

parmi 1675 fiches de maths

Désolé, votre version d'Internet Explorer est plus que périmée ! Merci de le mettre à jour ou de télécharger Firefox ou Google Chrome pour utiliser le site. Votre ordinateur vous remerciera !